Matrix of orthogonal projection

Click For Summary
An orthogonal projection matrix A satisfies the property A^2 = A, meaning that projecting a point twice onto the same line yields the same result as projecting it once. Geometrically, applying the projection twice does not change the outcome after the first projection. The computation method involves using the formula A = QQ^T, leading to A^2 = QQ^TQQ^T = QQ^T, confirming the geometric result. The discussion emphasizes understanding both the geometric and algebraic aspects of orthogonal projections. This reinforces the concept that repeated applications of an orthogonal projection do not alter the result after the initial projection.
morsel
Messages
29
Reaction score
0

Homework Statement


Let A be the matrix of an orthogonal projection. Find A^2 in two ways:
a. Geometrically. (consider what happens when you apply an orthogonal projection twice)
b. By computation, using the formula:
matrix of orthogonal projection onto V = QQ^T, where Q = [u1 ... um]


Homework Equations





The Attempt at a Solution


I have no idea how to approach (a).
(b). A^2 = Q Q^T Q Q^T = Q^2 (Q^T)^2

Thanks in advance.
 
Physics news on Phys.org
Let's start with (a).
Suppose we have 2 dimensions and A defines an orthogonal projection.
This means that any point v is projected orthogonally on a line.
What happens to this projection if we project it again on the line?
 
Last edited:
Does it go back to the original point?
 
morsel said:
Does it go back to the original point?

We can find on wikipedia a page http://en.wikipedia.org/wiki/Projection_%28linear_algebra%29" :

252px-Orthogonal_projection.svg.png

The transformation P is the orthogonal projection onto the line m.

and:

"In linear algebra and functional analysis, a projection is a linear transformation P from a vector space to itself such that P2 = P. "
 
Last edited by a moderator:
Question: A clock's minute hand has length 4 and its hour hand has length 3. What is the distance between the tips at the moment when it is increasing most rapidly?(Putnam Exam Question) Answer: Making assumption that both the hands moves at constant angular velocities, the answer is ## \sqrt{7} .## But don't you think this assumption is somewhat doubtful and wrong?

Similar threads

  • · Replies 23 ·
Replies
23
Views
10K
  • · Replies 1 ·
Replies
1
Views
2K
  • · Replies 2 ·
Replies
2
Views
3K
  • · Replies 6 ·
Replies
6
Views
4K
  • · Replies 9 ·
Replies
9
Views
2K
Replies
7
Views
2K
  • · Replies 1 ·
Replies
1
Views
2K
Replies
7
Views
2K
Replies
5
Views
2K
Replies
5
Views
3K